Matematică, întrebare adresată de ursucatalina47, 8 ani în urmă

[... ]>partea întreagă ​

Anexe:

Răspunsuri la întrebare

Răspuns de rezolvator1
1

Răspuns:

[\frac{1}{\sqrt{2} +\sqrt{1} } +\frac{1}{\sqrt{3} +\sqrt{2} } +...+\frac{1}{\sqrt{n+1} +\sqrt{n} }] =2\\

[\frac{\sqrt{2} -\sqrt{1}}{(\sqrt{2} +\sqrt{1})(\sqrt{2} -\sqrt{1})} +\frac{\sqrt{3} -\sqrt{2}}{(\sqrt{3} +\sqrt{2})(\sqrt{3} -\sqrt{2})} +...+\frac{\sqrt{n+1} -\sqrt{n}}{(\sqrt{n+1} +\sqrt{n})(\sqrt{n+1} -\sqrt{n})} ] =2

[\frac{\sqrt{2} -\sqrt{1}}{2-1} +\frac{\sqrt{3} -\sqrt{2}}{3-2} +...+\frac{\sqrt{n+1} -\sqrt{n}}{n+1-n} ] =2

[\sqrt{2} -\sqrt{1} +\sqrt{3} -\sqrt{2} +\sqrt{4} -\sqrt{3} +...+\sqrt{n} -\sqrt{n-1} +\sqrt{n+1} -\sqrt{n} ]=2\\

[\sqrt{n+1} -1 ]=2\\

2\leq \sqrt{n+1}-1  <3\\

3\leq \sqrt{n+1}  <4\\

⇔ 9 ≤ n+1 < 16

⇔ 8 ≤ n < 15

n ∈{8, 9, 10, 11, 12, 13, 14}

Alte întrebări interesante